Você está na página 1de 11

KYUNGPOOK Math J.

41(2001), 217-227

Lifting Idempotents and Projective Covers


Dinesh Khurana and R. N. Gupta
Centre for Advanced Study in Mathematics, Panjab University, Chandigarh-160014,
India.
e-mail : dkhurana@pu.ac.in, rng@pu.ac.in

(2000 Mathematics Subject Classification : 16D40)

A necessary and sufficient condition is given so that direct summands of a module,


with projective cover, have projective covers. We characterize the rings over which direct
summands of cyclic (respectively finitely generated) modules, with projective covers, have
projective covers. Some sufficient conditions are given so that direct summands of arbitrary
modules, with projective covers, have projective covers. Let L be a left ideal of a ring R.
It is said that idempotents lift modulo L in R if for every x ∈ R with x2 − x ∈ L there
exists an idempotent e in R such that e − x ∈ L. It is proved that if idempotents lift
modulo the Jacobson radical of a ring, then idempotents also lift modulo every one sided
ideal contained in the Jacobson radical. Let I be an ideal of a ring R. We prove that if
idempotents lift in R modulo every left ideal contained in I then, idempotents also lift in
R modulo every right ideal contained in I. On taking I = R we get that left exchange
rings are right exchange.

0. Introduction
Throughout rings are associative with unity, modules are unital left modules
and endomorphisms of modules act on the right.
Let U and M be two left R-modules. U is said to be M -projective in case for
each R-epimorphism g : M → N and each R-homomorphism γ : U → N there is
an R-homomorphism δ : U → M such that δg = γ. If U is M -projective for every
left R-module M then U is said to be projective. Projective left R-modules are just
direct summands of free left R-modules. A submodule N of a module M is said
to be superfluous or small (N ≪ M ) if for any submodule K of M , N + K = M
implies that K = M .
Dualizing the concept of an injective hull of a module, Bass [2] defined a pro-
jective cover of a module M to be an epimorphism p : P → M , such that P is
a projective module and Ker p ≪ P . Thus modules having projective covers are,
upto isomorphism, of the form P/K, where P is a projective module and K its
superfluous submodule. Unlike injective hulls, projective covers of modules seldom

(Received : July 10, 2000)


Key words and phrases : idempotents, projective modules, projective covers, lifting of
idempotents.

217
218 Dinesh Khurana and R. N. Gupta

exist. For instance, over a semiprimitive ring R (i.e., J(R) = 0) a module has a
projective cover if and only if it is already projective (see [4, 24.11(5)]). Let I be
an ideal of ring R and f be an idempotent of R/I. We say that f lifts modulo I in
R if there exists an idempotent e in R such that ē = e + I = f .
The following well known result gives a relationship between the concepts of
projective covers and lifting of idempotents.
Proposition 0.1.([1, Proposition 27.4]) Let R be a ring and I be a two sided
ideal of a ring R with I ≪ R R (or equivalently I ⊆ J(R)). Then the following
are equivalent :
(a) Idempotents lift modulo I ;
(b) Every direct summand of a left R-module R/I has a projective cover.
From Proposition 0.1 it follows that if I ⊆ J(R) is an ideal of a ring R such that
idempotents do not lift modulo I in R then, although the left R module R/I has
a projective cover, there exists a direct summand of left R-module R/I which does
not have a projective cover. More specifically, Let Q be the field of rational numbers
and R = {a/b ∈ Q : (6, b) = 1}. Then R is a ring with R/J(R) ∼ = R/2R⊕R/3R, but
neither R/2R nor R/3R have projective covers. So the natural question arises that
what exactly are the modules with projective covers whose every direct summand
also has a projective cover. One can easily prove the following
Lemma 0.2. Let p : P → M be a projective cover of R M with Ker p = K;
E = EndR (P ); S = {α ∈ E : K α ⊆ K} and T = {α ∈ E : P α ⊆ K}. For α ∈ S
define α̃ : M → M by (xp)α̃ = (xα)p, where x ∈ P . Then α̃ ∈ End(M ) and α → α̃
is a ring epimorphism S → End(M ) with kernel T . Thus θ : S/T → End(M )
defined as (ᾱ)θ = α̃ is an isomorphism.
Suppose that q : Q → M is also a projective cover of M with K1 = Ker q. Then
there exists an isomorphism γ : P → Q such that γq = p (see [1, Lemma 17.17]).
Let E1 = EndR (Q); S1 = {α ∈ E1 : K1 α ⊆ K1 } and T1 = {α ∈ S1 : Qα ⊆ K1 }.
One easily checks that the map δ : S → S1 defined as δ(σ) = γ −1 σγ, where σ is in
S, is an isomorphism with δ(T ) = T1 . Thus S and T are unique upto isomorphism.
Let M be as in Lemma 0.2 and N ⊆⊕ M . Fix an idempotent e in EndR (M ) such
that N = M e. There exists an idempotent ḡ in S/T such that θ(ḡ) = e. In section
1 we prove that N has a projective cover if and only if ḡ lifts to an idempotent
in S. We also prove that every direct summand of the left R-module M has a
projective cover if and only if every direct summand of the left S-module S/T has a
projective cover. Nicholson [5], defined left suitable rings and characterised them
as rings in which idempotents lift modulo every left ideal. In section 2 we study the
rings in which idempotents lift modulo every left ideal contained in the Jacobson
radical. We prove that these rings coincide with the rings in which idempotents lift
modulo the Jacobson radical. Using the results proved in section 1 we characterize
the rings in which idempotents lift modulo the Jacobson radical as rings over which
direct summands of cyclic left R-modules, with projective covers, have projective
Lifting Idempotents and Projective Covers 219

covers. It is proved that direct summands of finitely generated left R-modules,


with projective covers, have projective covers if and only if idempotents lift modulo
Mn (J) in Mn (R) for every natural number n.
In section 3 we give some sufficient conditions for direct summands of left R-
modules, with projective covers, to have projective covers. In particular, we prove
that if idempotents lift modulo the Jacobson radical of the endomorphism ring of
each free left R-module then if a left R-module has a projective cover, so does
its every direct summand. Warfield [8] proved that a left exchange ring is right
exchange as well. Nicholson [5] proved that left exchange rings coincide with left
suitable rings. In [7] Nicholson gave a simple proof of the fact that left exchange
rings coincide with right exchange rings. In section 4 we generalize this result and
prove that if I is a two sided ideal of a ring R such that idempotents lift in R
modulo every left ideal contained in I then idempotents also lift in R modulo every
right ideal contained in I.

1. Projective covers of direct summands


We first list some basic results which will often be referred to later.
Lemma 1.1.([1, Lemma 5.18]) If K ≪ M and f : M → N is a homomorphism
then Kf ≪ N . In particular, if K ≪ M ≤ N , then K ≪ N .
Lemma 1.2.([1, Proposition 17.11]) Let P be a projective left R-module with en-
domorphism ring E = EndR (P ). Let a ∈ E. Then

a ∈ J(E) ⇔ Im a ≪ P.

Lemma 1.3.([1, Exercise 17.15(2)]) Let M and N be two modules. If both M and
M ⊕ N have projective covers, then N also has a projective cover.
Lemma 1.4.([1, Lemma 17.17]) Suppose M has a projective cover p : P → M .
If Q is projective and q : Q → M is an epimorphism, then Q has a decomposition
Q = P ′ ⊕ P ′′ such that
(a) P ′ ∼
= P.
(b) P ′′ ⊆ Ker q.
(c) (q|P ′ ) : P ′ → M is a projective cover of M .
Lemma 1.5.([1, Exercise 27.1]) Let I be an ideal of R with I ⊆ J(R) and f be a
nonzero idempotent of R. If idempotents lift modulo I in R, then idempotents lift
modulo f If in f Rf .
Lemma 1.6.( [1, Lemma 27.2]) Let a left R-module M have a decomposition
M = M1 ⊕ ... ⊕ Mn such that each term Mi has a projective cover. Then an R-
homomorphism p : P → M is a projective cover if and only if P has a decomposition
220 Dinesh Khurana and R. N. Gupta

P1 ⊕ ... ⊕ Pn such that for each i = 1, ..., n

(p|Pi ) : Pi → Mi

is a projective cover.
Author got the idea of proving the following Theorem during personal communica-
tion with Professor Donald S. Passman.
Theorem 1.7. Let p : P → M be a projective cover of a left R-module M with
K = Ker p and E, S, T and θ be as in Lemma 0.2. Let M e, where e ∈ EndR (M )
is an idempotent with (ḡ)θ = e for some idempotent ḡ in S/T , be a direct summand
of M . Then the following are equivalent :
(a) There exists a direct summand P1 of P such that P1 p = M e ;
(b) M e has a projective cover ;
(c) There exists a decomposition P = P1 ⊕ P2 such that

P1 p = M e and P2 p = M (1 − e);

(d) ḡ lifts modulo T in S.


In particular, every direct summand of M has a projective cover if and only if
idempotents lift modulo T in S.
Proof. (a) ⇒ (b). As K ≪ P and P1 is a direct summand of P , Ker(p|P1 ) =
K ∩ P1 ≪ P1 (see Lemma 1.1). Thus (p|P1 ) : P1 → M e is a projective cover.
(b) ⇒ (c). As both M e and M = M e ⊕ M (1 − e) have projective covers,
M (1 − e) also has a projective cover by Lemma 1.3. Also as p : P → M is a
projective cover, by Lemma 1.6, there exists a decomposition P = P1 ⊕ P2 such
that
P1 p = M e and P2 p = M (1 − e);
(c) ⇒ (d). Let f ∈ E be an idempotent such that P1 = P f and P2 = P (1 − f ).

(1) P1 p = P f p = M e = M (ḡ)θ

and

(2) P2 p = P (1 − f )p = M (1 − e) = M (1 − g)θ

Now we show that f ∈ S. Let k ∈ K. Then

k = kf + k(1 − f ) ⇒ 0 = kp = kf p + k(1 − f )p

⇒ kf p ∈ M e ∩ M (1 − e) = 0 ⇒ kf ∈ Ker p = K
which in turn implies that f ∈ S. This in view of (1) and (2) gives that,

(3) M (ḡ)θ = P f p = (P p)(f¯)θ = M (f¯)θ


Lifting Idempotents and Projective Covers 221

and

(4) M (1 − g)θ = P (1 − f )p = (P p)(1 − f )θ = M (1 − f )θ.

Now by using (3) and (4) we get

(f¯)θ + (1 − f )θ = 1 ⇒ (ḡ)θ((f¯)θ + (1 − f )θ) = (ḡ)θ ⇒ (ḡ)θ(f¯)θ = (ḡ)θ.

Also

(ḡ)θ + (1 − g)θ = 1 ⇒ ((ḡ)θ + (1 − g)θ)(f¯)θ = (f¯)θ ⇒ (ḡ)θ(f¯)θ = (f¯)θ.

Thus (ḡ)θ = (f¯)θ = (ḡ)θ(f¯)θ and as θ is an isomorphism, ḡ = f¯.


(d) ⇒ (a). We may assume that g is an idempotent of S. Thus, P g is a direct
summand of P with

M e = M (ḡ)θ = (P p)(ḡ)θ = (P g)p.

Last assertion is obvious.


The following result improves upon [1, Proposition 27.4].
Corollary 1.8. Let I be an ideal of a ring R such that I ≪ R R. Let (R/I)ḡ,
where ḡ is an idempotent in R/I, be a direct summand of left R-module R/I. Then
the following are equivalent :
(a) ḡ lifts modulo I in R ;
(b) (R/I)ḡ has a projective cover.
Now we prove the last result of this section.
Proposition 1.9. Let p : P → M be a projective cover of a left R-module M with
K = Ker p and E, S, T and θ be as in Lemma 0.2. Let ḡ be an idempotent in S/T .
Then the direct summand M (ḡ)θ of the left R-module M has a projective cover if
and only if the direct summand (S/T )ḡ of the left S-module S/T has a projective
cover. In particular, every direct summand of the left R-module M has a projective
cover if and only if every direct summand of the left S-module S/T has a projective
cover.
Proof. Let α ∈ T . As Im α ⊆ K ≪ P , α is in J(E) by Lemma 1.2. Thus T is
contained in J(E) and so T ≪ S S. Thus the result follows from Theorem 1.7 and
Corollary 1.8.
2. Lifting idempotents
Let L be a left ideal of a ring R. Then I(L) = {r ∈ R : Lr ⊆ L}, called the idealizer
of L, is the largest subring of R in which L is a two sided ideal.
The following special case of Theorem 1.7 can be of independent interest as is
demonstrated in this section.
222 Dinesh Khurana and R. N. Gupta

Corollary 2.1. Let L be a superfluous left ideal of a ring R, I(L) be the idealizer
of L and θ : I(L)/L → EndR (R/L) be the canonical isomorphism. Let K be a
direct summand of left R-module R/L and ḡ is an idempotent in I(L)/L such that
K = (R/L)(ḡ)θ. Then the following are equivalent :
(a) ḡ lifts modulo L in I(L) ;
(b) (R/L)(ḡ)θ has a projective cover.
In particular, every direct summand of left R-module R/L has a projective cover if
and only if idempotents lift modulo L in I(L).
Nicholson [5] defined a ring R to be suitable if each of its elements satisfies the
equivalent properties of the following Proposition.
Proposition 2.2.(Nicholson [5, Proposition 1.1]) If R is a ring, the following
conditions are equivalent for an element x of R :
(a) There exists an idempotent e in R with e − x ∈ R(x − x2 ) ;
(b) There exists an idempotent e in Rx and an element c ∈ R such that
(1 − e) − c(1 − x) ∈ J(R) ;
(c) There exists an idempotent e in Rx such that R = Re + R(1 − x) ;
(d) There exist an idempotent e in Rx such that 1 − e ∈ R(1 − x).
Nicholson [5, Corollary 1.3] proved that a ring R is suitable if and only if idem-
potents lift modulo every one-sided ideal of R. We are interested in rings in which
idempotents lift modulo every one-sided ideal contained in the Jacobson radical.
In Theorem 2.4 below, we prove that such rings coincide with the rings in which
idempotents lift modulo the Jacobson radical. First we prove the following
Lemma 2.3. For a left ideal L of a ring R the following are equivalent :
(a) Idempotents lift modulo every left ideal contained in L ;
(b) Every element x ∈ R such that x2 − x ∈ L satisfies the following equivalent
conditions :
(i) There exists an idempotent e in R with e − x ∈ R(x − x2 ) ;
(ii) There exists an idempotent e in Rx and an element c ∈ R such that
(1 − e) − c(1 − x) ∈ J(R) ;
(iii) There exists an idempotent e in Rx such that R = Re + R(1 − x) ;
(iv) There exist an idempotent e in Rx such that 1 − e ∈ R(1 − x).
Proof. The four properties listed under (b) are equivalent by Proposition 2.2.
(a) ⇒ (b)(i). Let x ∈ R be such that x2 − x ∈ L. As R(x2 − x) ⊆ L is a left
ideal of R so by (a) there exists an idempotent e ∈ R such that e − x ∈ R(x2 − x).
(b)(i) ⇒ (a). Let T ⊆ L be a left ideal of R and x2 − x ∈ T for some x ∈ R. By
(b)(i) there exists an idempotent e ∈ R such that e − x ∈ R(x2 − x) ⊆ T . This
completes the proof.
Theorem 2.4. Let R be a ring with J = J(R). If idempotents lift modulo J in R
then idempotents lift in R modulo every left ideal contained in J.
Proof. Let x ∈ R be such that x2 − x ∈ J. In view of Lemma 2.3 we have to
Lifting Idempotents and Projective Covers 223

show that there exists an idempotent e ∈ Rx and an element c ∈ R such that


(1 − e) − c(1 − x) ∈ J.
Now as idempotents lift modulo J in R so there exists an idempotent f ∈ R
such that f − x ∈ J. So u = 1 − f + x is an invertible element with ū = 1̄ in R/J.
Let e = u−1 f u = u−1 f (1 − f + x) = u−1 f x. Clearly e is an idempotent in Rx and
ē = f¯ = x̄. Thus x − e = (1 − e) − 1(1 − x) ∈ J.
It is not known whether the lifting of idempotents modulo the Jacobson radical in
rings is categorical.
Remark 2.5. Suppose that idempotents lift modulo a left ideal L ⊆ J(R) in R.
On the same lines one can prove that idempotents lift in R modulo every left ideal
contained in L also. Note that this does not hold for left ideals which are not con-
tained in the Jacobson radical. For example, idempotents lift modulo 4Z in Z but
do not lift modulo 12Z in Z, where Z denotes the ring of integers.
The following result characterizes the rings over which direct summands of cyclic
modules, with projective covers, have projective covers.
Theorem 2.6. The following conditions are equivalent for a ring R with J =
J(R):
(a) Idempotents lift modulo J in R ;
(b) Idempotents lift modulo L in I(L) for every left ideal L ⊆ J(R) ;
(c) For any left ideal L ⊆ J(R) every direct summand of left R-module R/L has
a projective cover ;
(d) Every direct summand of a cyclic left R-module, with projective cover, has
a projective cover.
Proof. (a) ⇒ (b). Let L ⊆ J be a left ideal of R and x2 − x ∈ L for some x ∈ I(L).
By Theorem 2.4 there exists an idempotent e ∈ R such that e − x ∈ L. This implies
that e ∈ I(L).
(b) ⇒ (c). This follows from Corollary 2.1.
(c) ⇒ (d). Let M be a cyclic left R-module with projective cover. By Lemma
1.4, M ∼ = Re/L for some idempotent e ∈ R and left ideal L ⊆ Je. Now
(Re/L) ⊕ (R(1 − e)/J(1 − e)) ∼ = R/(L ⊕ J(1 − e)). As L1 = L ⊕ J(1 − e) ⊆ J
is a left ideal of R so, by (c), every direct summand of R/L1 has a projective cover.
Now as direct summands of M ∼ = Re/L are isomorphic to direct summands of R/L1
so every direct summand of Re/L has a projective cover.
(d) ⇒ (a). As every direct summand of left R-module R/J has a projective
cover so by Corollary 1.8 idempotents lift modulo J in R.
Remark 2.7. From Remark 2.5 and the proof of Theorem 2.6 it is clear that if
idempotents lift modulo an ideal I ⊆ J(R) of R and L ⊆ I is a left ideal of R then
every direct summand of left R-module R/L has a projective cover.
Lemma 2.8. Let M be an n-generated left R module. Then M has a projective
cover if and only if M ∼
= R(n) e/L, where e ∈ EndR (R(n) ) ∼
= Mn (R) is an idempo-
224 Dinesh Khurana and R. N. Gupta

tent and L ⊆ J (n) e is a submodule of left R-module R(n) e.


Proof. As M is n-generated, there exists an epimorphism p : R(n) → M . By
Lemma 1.4 there exists an idempotent e ∈ EndR (R(n) ) ∼ = Mn (R) such that
(p|R(n) e) : R(n) e → M is a projective cover. Let L = Ker (p|R(n) e). Then
M∼ = R(n) e/L with L ≪ R(n) e. Thus L ⊆ Rad(R(n) e) = J (n) e.
In the following result we give a characterization of the rings over which direct sum-
mands of finitely generated modules, with projective covers, have projective covers.

Theorem 2.9. For a ring R with J = J(R) the following conditions are equiva-
lent :
(a) Idempotents lift modulo Mn (J) in Mn (R) for every natural number n ;
(b) Every direct summand of any finitely generated left R-module, with projec-
tive cover, has a projective cover.
Proof. (b) ⇒ (a). This is well known (see e.g., [1, Exercise 27.2]).
(a) ⇒ (b). Let R M be a finitely generated module with projective cover. By
Lemma 2.8 M ∼ = R(n) e/L, where e is an idempotent in E ′ = EndR (R(n) ) ∼
= Mn (R)
and L is a superfluous submodule of left R-module R(n) e. Let E = EndR (R(n) e) ∼
=
eMn (R)e, S = {α ∈ E : Lα ⊆ L} and T = {α ∈ E : R(n) eα ⊆ L}. In view of
Theorem 1.7, it is enough to show that idempotents lift modulo T in S.
Let S ′ = {α ∈ E ′ : Lα ⊆ L} and T ′ = {α ∈ E ′ : R(n) α ⊆ L}. Then T ′ is a left
ideal of E ′ and a two sided ideal of S ′ . Also as Le = L, e ∈ S ′ . As L ≪ R R(n) e,
by Lemma 1.1, L ≪ R R(n) and thus, by Lemma 1.2, T ′ ⊆ J(E ′ ). So T ′ ⊆ J(S ′ ).
Claim: S = eS ′ e and T = eT ′ e.
As e ∈ S ′ , eS ′ e ⊆ S ′ . So if β ∈ S ′ then Leβe ⊆ L showing that eβe ∈ S. Con-
versely, if eαe ∈ S, where α ∈ Mn (R), then eαe ∈ S ′ also. Thus eαe ∈ eS ′ e showing
that S = eS ′ e.
Also as e ∈ S ′ , eT ′ e ⊆ T ′ . Let γ ∈ T ′ . Then R(n) e(eγe) = R(n) eγe ⊆ L and so
eγe ∈ T . Conversely, if eαe ∈ T , where α ∈ Mn (R), then R(n) eαe = R(n) e(eαe) ⊆
L. Thus eαe ∈ T ′ . This establishes the claim.
Let I(T ′ ) denote the idealizer of T ′ in E ′ . Obviously, S ′ ⊆ I(T ′ ). Conversely,
suppose that α ∈ I(T ′ ). Then by definition T ′ α ⊆ T ′ and so R(n) T ′ α ⊆ L.
Now as R(n) is a generator in the category of left R-modules, R(n) T ′ = L. So
R(n) T ′ α = Lα ⊆ L implying that α ∈ S ′ . Thus S ′ = I(T ′ ). As idempotents lift
modulo J(E ′ ) in E ′ (by (a)) and T ′ ⊆ J(E ′ ) is a left ideal of E ′ , by Theorem 2.6,
idempotents lift modulo T ′ in S ′ = I(T ′ ). Now as e ∈ S ′ and T ′ ⊆ J(S ′ ), by
Lemma 1.5, idempotents lift modulo T = eT ′ e in S = eS ′ e.

3. Some sufficient conditions


In view of Theorem 2.6 and Theorem 2.9, one asks that does there exist a char-
Lifting Idempotents and Projective Covers 225

acterization of rings over which if a module has a projective cover then so does its
every direct summand. In this section we give some sufficient conditions for this to
happen.
Proposition 3.1. Let p : P → M be a projective cover such that idempotents lift
modulo the Jacobson radical of EndR (P ) and P generates L = Ker p. Then every
direct summand of M has a projective cover.
Proof. Let E = End(P ), S = {α ∈ E : Lα ⊆ L} and T = {α ∈ E : P α ⊆ L}. In
view of Theorem 1.7 we have to show that idempotents lift modulo T in S. As T
is two sided ideal in S so S ⊆ I(T ), the idealizer of T in E. Conversely suppose
that α ∈ I(T ). Then T α ⊆ T and so P T α ⊆ L. But as P generates L so P T = L
and thus Lα ⊆ L. Hence I(T ) ⊆ S and thus I(T ) = S. Now as idempotents lift
modulo J(E) in E so, by Theorem 2.6, idempotents lift modulo T in S.
Proposition 3.2. Let M be a left R-module with projective cover and F be a free
left R-module such that F maps onto M and idempotents lift modulo the Jacobson
radical of EndR (F ). Then every direct summand of M has a projective cover.
Proof. By Lemma 1.4 M = ∼ F e/L, where e ∈ E ′ = EndR (F ) is an idempo-
tent and L ≪ R F e. By hypothesis idempotents lift modulo J(E ′ ) in E ′ . Let
S ′ = {α ∈ E ′ : Lα ⊆ L} and T ′ = {α ∈ E ′ : F α ⊆ L}. Now as in the proof of
Theorem 2.9, it can be seen that idempotents lift modulo eT ′ e in eS ′ e.
Let E = EndR (F e), S = {α ∈ E : Lα ⊆ L} and T = {α ∈ E : F eα ⊆ L}. It
can easily be checked that S = eS ′ e and T = eT ′ e. So idempotents lift modulo T
in S. Thus by Theorem 1.7 every direct summand of M has a projective cover.
Corollary 3.3. Let R be a ring such that idempotents lift modulo the Jacobson
radical of endomorphism ring of every free left R-module. Then every direct sum-
mand of any left R-module, with projective cover, has a projective cover.
Recall that a ring is called left perfect if every left R-module has a projective
cover.

Since over semiprimitive rings only projective modules have projective covers,
direct summands of left modules, with projective covers, over semiprimitive or left
perfect rings, have projective covers. In both these cases idempotents lift modulo
the Jacobson radical of endomorphism ring of every free left R-module (see [5]).
Also if direct summands of left R-modules, with projective covers, have projective
covers then idempotents lift modulo the Jacobson radical of endomorphism ring of
every finitely generated free left R-module (see Theorem 2.9). This makes one ask
the following question.
Question 3.4. Let R be a ring such that every direct summand of any left R-
module, with projective cover, has a projective cover. Then do idempotents lift
modulo the Jacobson radical of endomorphism ring of every free left R-module?
226 Dinesh Khurana and R. N. Gupta

4. Applications
We now give some applications of the results proved above.
Lemma 4.1.([4, Proposition 21.20]) Let e, f be idempotents in a ring R. Then
the following are equivalent :
(a) eR ∼= f R as right R-modules ;
(b) Re ∼= Rf as left R-modules ;
(c) There exist elements a, b ∈ R such that ab = e and ba = f .
The following result improves upon [7, Proposition and Corollary] and the proof
runs on the similar lines.
Proposition 4.2. Let x be an element of a ring R. If there exists an idempotent
e ∈ Rx such that (1 − e) ∈ R(1 − x), then there exists an idempotent e′ ∈ xR such
that 1 − e′ ∈ (1 − x)R. Moreover, if R R has the internal cancellation property, then
there exists a unit u in R such that e′ = ueu−1
Proof. Suppose e = rx and 1 − e = s(1 − x). We may assume that r ∈ eR and
s ∈ (1 − e)R. Thus

(1) rxr = r, s(1 − x)s = s, rxs = 0 and s(1 − x)r = 0.

Define r′ = 1 − (1 − x)s + (1 − x)r. Then

r′ x = (1 − (1 − x)s)x + (1 − x)rx
(2) = (1 − (1 − x)s)x + (1 − x)(1 − s(1 − x))
= (1 − (1 − x)s)x + (1 − (1 − x)s)(1 − x) = 1 − (1 − x)s.

By using (1), we have sr′ = s − s(1 − x)s + s(1 − x)r = 0 and so

(3) r′ xr′ = r′ − (1 − x)sr′ = r′ .

Thus e′ = xr′ is an idempotent and

1 − e′ = 1 − xr′ = 1 − x(1 − (1 − x)s + (1 − x)r)


= 1 − x + x(1 − x)s − x(1 − x)r
= (1 − x) + (1 − x)xs − (1 − x)xr ∈ (1 − x)R

Now suppose R R has internal cancellation property. As 1 − e = s(1 − x) and


s(1 − x)s = s, by Lemma 4.1, R(1 − e) ∼ = R(1 − x)s. So, by hypothesis and
Lemma 4.1, Re ∼= R(1 − (1 − x)s) = Rr′ x ∼ = Rxr′ = Re′ (see (2) and (3)). Thus
∼ ′
R(1 − e) = R(1 − e ) and so there exists a desired unit element u (see [4, Exercise
21.15]).
Proposition 4.3. For a two sided ideal of a ring R the following are equivalent :
(a) Idempotents lift in R modulo every left ideal contained in I ;
(b) Idempotents lift in R modulo every right ideal contained in I.
Lifting Idempotents and Projective Covers 227

Proof. Follows from Proposition 4.2 and Lemma 2.3.


Proposition 4.5. Let R be a ring in which idempotents lift modulo J(R) and x̄
be a nonzero idempotent in R/J(R). Then Rx contains a non-zero idempotent.
Proof. By Lemma 2.3 there exists an idempotent e ∈ Rx such that 1−e ∈ R(1−x).
Now e = 0 ⇒ 1 ∈ R(1 − x) ⇒ x ∈ R(x − x2 ) ⊆ J(R). Thus e ̸= 0 and the proof is
complete.
Corollary 4.6.(Nicholson [6, Proposition 1.4]) Let R be a ring in which idempo-
tents lift modulo J(R) and every non-zero left ideal of R/J(R) contains a non-zero
idempotent. Then every left ideal of R, not contained in J(R), contains a non-zero
idempotent.
Proof. Let x ∈ R\J(R). Let rx be a non-zero idempotent in (R/J)x̄. By Propo-
sition 4.5 Rrx ⊆ Rx contains a non-zero idempotent.

References
[1] F. W. Anderson and K. R. Fuller, Rings and Categories of Modules, 13, Springer-
Verlag, 2nd edition, (1992).
[2] Hyman Bass, Finitistic dimension and a homological generalization of semiprimary
rings, Trans. Amer. Math. Soc., 95(1960), 466-488.
[3] E. G. Evans Jr., Krull-Schmidt and cancellation over local rings, Pacific J. Math.,
46(1973), 115-121.
[4] T. Y. Lam, A First Course in Noncommutative Rings, Springer-Verlag, GTM 131,
(1991).
[5] W. K. Nicholson, Lifting idempotents and exchange rings, Trans. Amer. Math. Soc.,
229(1977), 269-278.
[6] W. K. Nicholson, I-Rings, Trans. Amer. Math. Soc., 207(1975), 361-373.
[7] W. K. Nicholson, On exchange rings, Comm. Alg., 25(1997), 1917-1918.
[8] R. B. Warfield Jr., Exchange rings and decompositions of modules, Math. Ann.,
199(1972), 31-36.

Você também pode gostar